Difference between revisions of "Talk:2011 AIME I Problems/Problem 12"

(Mistake: new section)
Line 2: Line 2:
  
 
denominator: n+3
 
denominator: n+3
 +
 +
==Solution is incorrect==
 +
 +
I don't see where <math>\dbinom{n+3}{3}</math> or <math>\dbinom{n+2}{2}</math> came from.  I'm getting <math>\dbinom{n+1}{3}</math> and <math>\dbinom{n+1}{2}</math>, respectively.  Case 2 is identical to cases 3 and 4.  In case 1, the number of women is still the same, only that we are adding more dividers.  The number of possible dividers that could potentially be added is the same.
  
 
== Mistake ==
 
== Mistake ==
  
 
If you actually solve the inequality at the end by hand with the quad. formula, you get 593 as the answer.
 
If you actually solve the inequality at the end by hand with the quad. formula, you get 593 as the answer.
 +
 +
The solution itself is incorrect - hesa57

Revision as of 11:55, 2 March 2016

I think in the solution of this problem some cases are repeated and correct answer

denominator: n+3

Solution is incorrect

I don't see where $\dbinom{n+3}{3}$ or $\dbinom{n+2}{2}$ came from. I'm getting $\dbinom{n+1}{3}$ and $\dbinom{n+1}{2}$, respectively. Case 2 is identical to cases 3 and 4. In case 1, the number of women is still the same, only that we are adding more dividers. The number of possible dividers that could potentially be added is the same.

Mistake

If you actually solve the inequality at the end by hand with the quad. formula, you get 593 as the answer.

The solution itself is incorrect - hesa57